Five patients undergo surgery for various reasons. Each patient has a particular prior medical history. Which of the following patients would be most problematic to the anesthesiologist if thiopental is used during the surgery?

Questions 171

ATI RN

ATI RN Test Bank

Chemotherapy drugs Questions

Question 1 of 5

Five patients undergo surgery for various reasons. Each patient has a particular prior medical history. Which of the following patients would be most problematic to the anesthesiologist if thiopental is used during the surgery?

Correct Answer: C

Rationale: Thiopental, a barbiturate, can cause bronchospasm and respiratory depression, which is particularly risky in a patient with asthma, potentially exacerbating airway issues.

Question 2 of 5

A 58-year-old woman with end-stage cervical cancer has undergone surgery, chemotherapy, and radiation therapy. She has chronic pelvic pain and is being treated with a fentanyl patch. Because of continued intractable pain, she places three patches on at the same time. She is found dead 6 h later by her caretaker. What is the most likely explanation for her death?

Correct Answer: C

Rationale: Fentanyl overdose from multiple patches causes respiratory depression (hypoventilation) by acting on opioid receptors in the brainstem, leading to fatal suppression of breathing.

Question 3 of 5

A 27-year-old man presents to the urgent care clinic with multiple painful ulcers on the shaft of his penis. Some strains of the causative virus are resistant because of a mutated thymidine kinase enzyme. Which of the following antivirals may be less effective in treating this man's infection if his strain of virus has a mutated thymidine kinase?

Correct Answer: A

Rationale: Acyclovir requires viral thymidine kinase for activation; a mutated enzyme in herpes simplex virus (HSV) reduces its effectiveness, unlike foscarnet, which acts directly on viral DNA polymerase.

Question 4 of 5

A 45-year-old man with an elevated bleeding time is scheduled to undergo an elective hernia repair in approximately 12 h. He is given vitamin K orally and the bleeding time is still elevated. What is the most likely explanation for this finding?

Correct Answer: B

Rationale: Vitamin K corrects bleeding due to deficiency (e.g., warfarin use) by enabling synthesis of new clotting factors (II, VII, IX, X), which takes hours to days, not immediate effect within 12 hours.

Question 5 of 5

A 3-year-old girl presents to the emergency department with a history of recurrent UTIs with costovertebral angle tenderness, high fever, and dysuria. A urine culture grows gram-negative lactose-fermenting rods. The physician suspects E. coli pyelonephritis. Ciprofloxacin is highly effective against E. coli in vitro, but the physician chooses not to use it in this case. Why would she choose not to prescribe ciprofloxacin?

Correct Answer: B

Rationale: Ciprofloxacin is contraindicated in children (<18 years) due to the risk of cartilage damage and impaired growth, despite its efficacy against E. coli.

Access More Questions!

ATI RN Basic


$89/ 30 days

ATI RN Premium


$150/ 90 days

Similar Questions